Parametrizing a Triangle in 3D Space: What is the Domain of r(u,v)?

Click For Summary
SUMMARY

The discussion focuses on parametrizing a triangle defined by points P=(1,0,0), Q=(0,2,0), and R=(0,0,2) in 3D space. The triangle is represented by the plane equation 2x+y+z+2=0, and the parametrization is given as r(u,v) = u*PQ + v*PR + P. The domain of r(u,v) is determined to be u ∈ [0, 1-v] and v ∈ [0, 1]. Participants emphasize the importance of visualizing the uv coordinate system to establish the limits for the double integral.

PREREQUISITES
  • Understanding of 3D geometry and triangle representation
  • Familiarity with parametrization techniques in vector calculus
  • Knowledge of double integrals and their applications in area calculations
  • Ability to interpret and manipulate plane equations in 3D space
NEXT STEPS
  • Learn about visualizing parametrizations in 3D space using coordinate systems
  • Study the application of double integrals in calculating areas of irregular shapes
  • Explore the relationship between plane equations and their geometric representations
  • Investigate advanced parametrization techniques for complex surfaces
USEFUL FOR

Students in mathematics, particularly those studying calculus and geometry, as well as educators and professionals involved in 3D modeling and computer graphics.

Nikitin
Messages
734
Reaction score
27

Homework Statement


A triangle is defined by the 3 points P=(1,0,0), Q=(0,2,0) and R=(0,0,2).

Set up the double integral over its area.

The Attempt at a Solution



The triangle can be described as a plane 2x+y+z+2=0, with xE [0,1], yE [0,2], zE [0,2].

I parametrized it into r(u,v) =u*PQ+v*PR+P = u[-1,2,0]+v[-1,0,2]+(1,0,0).

But what's the domain of r(u,v)? I think it is uE [0,1-v], vE[0,1], but even if it is I found it using only gut feeling.
 
Physics news on Phys.org
Nikitin said:

Homework Statement


A triangle is defined by the 3 points P=(1,0,0), Q=(0,2,0) and R=(0,0,2).

Set up the double integral over its area.

The Attempt at a Solution



The triangle can be described as a plane 2x+y+z+2=0, with xE [0,1], yE [0,2], zE [0,2].

I parametrized it into r(u,v) =u*PQ+v*PR+P = u[-1,2,0]+v[-1,0,2]+(1,0,0).

But what's the domain of r(u,v)? I think it is uE [0,1-v], vE[0,1], but even if it is I found it using only gut feeling.

What you need to do is draw a ##uv## coordinate system. Then mark which ##uv## points are mapped to which points on your plane. For example ##(u,v)=(0,0)## goes to ##(1,0,0)##. Mark also where ##(u,v) = (1,0)## and ##(u,v)=(0,1)## go. This will give you a picture of your ##uv## domain and you will be able to write your double integral limits for ##u## and ##v##.
 
Could you explain the general strategy in further detail? Why did you pick (u,v) = (0,0), (1,0) and (0,1), for instance? Is it because those are the minimal and maximal values of u and v?

I tried to draw the u,v plane, but I wasn't sure what to do...
 
Nikitin said:
Could you explain the general strategy in further detail? Why did you pick (u,v) = (0,0), (1,0) and (0,1), for instance? Is it because those are the minimal and maximal values of u and v?

I tried to draw the u,v plane, but I wasn't sure what to do...

Just draw two perpendicular axes and label them u and v instead of x and y.

If you look at your parameterization, r(u,v) =u*PQ+v*PR+P = u[-1,2,0]+v[-1,0,2]+(1,0,0) it is obvious that that those values of u and v give the corners of your triangle. Joining those the points (u,v) = (0,0), (1,0) and (0,1) in the uv plane gives you a picture of the uv domain.

It is just like as if you had written the xyz equation of the plane as z = 2-2x-y and used x and y as your parameters. Then you would have used the domain triangle in the xy plane for your xy limits.
 
hmm, OK. Thanks
 
Question: A clock's minute hand has length 4 and its hour hand has length 3. What is the distance between the tips at the moment when it is increasing most rapidly?(Putnam Exam Question) Answer: Making assumption that both the hands moves at constant angular velocities, the answer is ## \sqrt{7} .## But don't you think this assumption is somewhat doubtful and wrong?

Similar threads

  • · Replies 2 ·
Replies
2
Views
2K
Replies
11
Views
4K
Replies
2
Views
2K
  • · Replies 3 ·
Replies
3
Views
7K
  • · Replies 3 ·
Replies
3
Views
4K
Replies
3
Views
2K